CBSE Questions for Class 8 Maths Rational Numbers Quiz 1 - MCQExams.com

State whether the statements are true (T) or false (F).
The rational numbers $$\dfrac{1}{2}$$ and $$-\dfrac{5}{2}$$ are on the opposite sides of $$0$$ on the number line.
  • True
  • False
State whether the statements are true (T) or false (F).
The rational numbers can be represented on the number line.
  • True
  • False
Solve following equation-
$$\dfrac { 3 }{ 7 } +\left( \dfrac { -6 }{ 11 }  \right) +\left( \dfrac { -8 }{ 21 }  \right) +\left( \dfrac { 5 }{ 22 }  \right)$$
  • $$\dfrac { -125 }{ 462 }$$
  • $$\dfrac { 125 }{ 462 }$$
  • $$\dfrac { 462 }{ 125 }$$
  • $$\dfrac { -462 }{ 125 }$$
Find multiplicative inverse of the following-
$$\dfrac { -5 }{ 8 } \times \dfrac { -3 }{ 7 }$$
  • $$\dfrac { 15 }{ 56 }$$
  • $$\dfrac { 56 }{ 15 }$$
  • $$\dfrac { -15 }{ 56 }$$
  • $$\dfrac { -56 }{ 15 }$$
Name which property is used in following operation-
$$\dfrac { 1 }{ 2 } \times \left( 5\times \dfrac { 2 }{ 5 }  \right) =\left( \dfrac { 1 }{ 2 } \times 5 \right) \times \dfrac { 2 }{ 5 }$$
  • Associative
  • Reciprocal
  • Commutative
  • Additive inverse
Multiply $$\dfrac { 6 }{ 13 }$$ by the reciprocal of $$\dfrac { -7 }{ 16 }$$
  • $$\dfrac { -91 }{ 96 }$$
  • $$\dfrac { 96 }{ 91 }$$
  • $$\dfrac { 91 }{ 96 }$$
  • $$\dfrac { -96 }{ 91 }$$
$$\displaystyle \frac{-7}{5} + \left(\displaystyle \frac{2}{-11} + \frac{-13}{25} \right) = \left(\displaystyle \dfrac{-7}{5} + \frac{2}{-11} \right) + \frac{-13}{25}$$
This property is
  • closure
  • commutative
  • associative
  • identity
$$x$$ is additive identity of rational numbers.Then the value of x is
  • 0
  • 1
  • -1
  • None
For any rational number $$\dfrac {a}{b}$$, its additive inverse is ..........
  • $$\dfrac {a}{b}$$
  • $$\dfrac {b}{a}$$
  • $$\dfrac {-b}{a}$$
  • $$\dfrac {-a}{b}$$
State true or false.
The three rational number between $$5$$ and $$6$$ are $$ \displaystyle\frac{21}{4},\frac{22}{4},\frac{23}{4}$$.
  • True
  • False
Choose the correct option for the following statement.
The property allows you to compute $$\displaystyle \frac{1}{3}\times \left ( 6 \times\frac{4}{3} \right )as \left ( \frac{1}{3} \times 6\right )\times \frac{4}{3}$$ is Associativity.
  • The given statement is true.
  • The given statement is false.
  • Incomplete information
  • None of these
State the following statement is True or False
The reciprocal of $$0$$ lie on the number line.
  • True
  • False
The property under multiplication used is:
$$-\dfrac{13}{17}\times\dfrac{-2}{7}=\dfrac{-2}{7}\times \dfrac{-13}{17}$$
  • Associative Property
  • Commutative Property
  • Distributive Property
  • Identity Property
Two rational numbers between $$\dfrac{2}{3}$$ and $$\dfrac{5}{3}$$ are :
  • $$\dfrac{1}{6}$$ and $$\dfrac{2}{6}$$
  • $$\dfrac{1}{2}$$ and $$\dfrac{2}{1}$$
  • $$\dfrac{5}{6}$$ and $$\dfrac{7}{6}$$
  • $$\dfrac{2}{3}$$ and $$\dfrac{4}{3}$$
Between any two rational numbers, 
  • there is no rational number
  • there is exactly one rational number
  • there are infinitely many rational numbers
  • there are only rational numbers and no irrational numbers
The property under multiplication used in each of the following.

$$\dfrac{-4}{5}\times 1 = 1 \times\dfrac{-4}{5}= -\dfrac{4}{5}$$
  • Commutative Property
  • Associative Property
  • Distributive Property
  • Identity Property
The property under multiplication used in each of the following is the 
$$\dfrac{-19}{29}\times\dfrac{29}{-19}=1$$
  • Commutative Property
  • Multiplicative inverse
  • Associative Property
  • Indentity Property
A rational number between $$\displaystyle \frac{1}{4}$$ and $$\displaystyle \frac{1}{3}$$ is
  • $$\displaystyle \frac{7}{24}$$
  • $$\displaystyle \frac{16}{48}$$
  • $$\displaystyle \frac{13}{48}$$
  • All the above
Find the five rational numbers between  $$\displaystyle \frac{1}{2}$$ and $$\displaystyle \frac{3}{2}$$
  • $$0.5 < 0.6 < 0.7 < 0.8 ... < 1.1 < ... < 1.15 < 1.50$$
  • $$0.5 < 0.6 < 1.7 < 3.8 ... < 1.8< ... < 1.15 < 1.50$$
  • $$0.5 < 0.6 < 0.7 < 2.8 ... < 1.1 < ... < 1.15 < 1.50$$
  • $$0.5 < 0.6 < 0.7 < 0.8 ... < 3.1 < ... < 1.15 < 1.50$$
State true or false:
The product of two rational numbers is always a rational number.
  • True
  • False
If D be subset of `the set of all rational numbers, which can be expressed as terminating decimals, then D is closed under the binary operations of
  • addition, subtraction and division.
  • addition, multiplication and division.
  • addition, subtraction and multiplication.
  • subtraction, multiplication and division.
Choose the rational number which  lie between rational numbers $$-\dfrac{2}{5}$$ and $$-\dfrac{1}{5}$$.
  • $$-\dfrac{1}{4}$$
  • $$-\dfrac{3}{10}$$
  • $$\dfrac{3}{10}$$
  • $$-\dfrac{7}{20}$$
The additive inverse of $$\displaystyle \frac{-a}{b}$$ is
  • $$\displaystyle \frac{a}{b}$$
  • $$\displaystyle \frac{b}{a}$$
  • $$\displaystyle \frac{-b}{a}$$
  • $$\displaystyle \frac{-a}{b}$$
Which one of the following is the rational number lying between $$\displaystyle \frac{6}{7} \ and \ \frac{7}{8}?$$
  • $$\displaystyle \frac{3}{4}$$
  • $$\displaystyle \frac{99}{122}$$
  • $$\displaystyle \frac{95}{112}$$
  • $$\displaystyle \frac{97}{112}$$
Multiplicative inverse of '0' is
  • $$\displaystyle \frac{1}{0}$$
  • 0
  • Does not exist
  • $$\displaystyle \frac{0}{0}$$
Write the additive inverse of each of the following: $$\dfrac{2}{8}$$ and $$-\dfrac{5}{9}$$
  • $$-\dfrac{2}{8}\;;\; \dfrac{5}{9}$$.
  • $$-\dfrac{1}{5};\dfrac{3}{5}$$
  • $$\dfrac{1}{8};\dfrac{5}{9}$$
  • $$\dfrac17;\dfrac89$$
Write the additive inverse of each of the following rational numbers:
$$\displaystyle\frac{4}{9}$$; $$\displaystyle\frac{-13}{7}$$; $$\displaystyle\frac{5}{-11}$$; $$\displaystyle\frac{-11}{-14}$$
  • $$\displaystyle\frac{-4}{9};\;\displaystyle\frac{13}{7};\;\displaystyle\frac{5}{11};\;\displaystyle\frac{-13}{14}$$
  • $$\displaystyle\frac{-4}{9};\;\displaystyle\frac{1}{7};\;\displaystyle\frac{5}{11};\;\displaystyle\frac{-11}{14}$$
  • $$\displaystyle\frac{-4}{9};\;\displaystyle\frac{13}{7};\;\displaystyle\frac{5}{11};\;\displaystyle\frac{-11}{14}$$
  • $$\displaystyle\frac{-5}{9};\;\displaystyle\frac{13}{7};\;\displaystyle\frac{5}{11};\;\displaystyle\frac{-11}{14}$$
Which of the following statement is true about a rational number $$\displaystyle \frac{-2}{3}$$ ?
  • It lies to the left side of $$'0'$$ on the number line.
  • It lies to the right side of $$'0'$$ on the number line.
  • It is not possible to represent on the number line.
  • It cannot be determined on which side the number lies.
Rational numbers are closed under substraction.
  • True
  • False 
  • Cannot be determined
  • None
What is the additive inverse of $$\displaystyle\frac{a}{b}$$?
  • $$\dfrac{-a}{b}$$
  • $$\dfrac{b}{a}$$
  • $$\dfrac{-b}{a}$$
  • $$\dfrac{a}{b}$$
0:0:1


Answered Not Answered Not Visited Correct : 0 Incorrect : 0

Practice Class 8 Maths Quiz Questions and Answers